Advanced Surgery Flashcards

1
Q

A 48-year-old patient was operated on 1.5 months ago for chronic calculous cholecystitis. Endoscopic cholecystectomy was performed. Before the operation she was ill for about 6 years. There was an episode of short-term jaundice, which was stopped independently.
Currently she complains of dull moderate intensity pain in the right hypochondrium for 6-7 days, jaundice of the sclera and discoloration of the stool in the last 2 days.
Objectively: the condition is satisfactory, the pulse is 80 beats / min, blood pressure is 145/80 mm Hg, the skin is of normal color, the sclera are icteric. The abdomen is not swollen, the liver is on the edge of the costal arch, the edge is sharp, slightly painful on palpation. The temperature is 37.2 C.
On ultrasound, it was found that the liver is of normal echogenicity, the gallbladder is not visualized.The intrahepatic bile ducts are dilated, the choledoch is 15 mm in diameter.
Task:
1. Assume the most likely diagnosis.
2. What additional examination is necessary, assume the results.
3. Make a differential diagnostic series
4. Determine the patient’s further management tactics
5. List possible complications in case of inadequate treatment

A

1.Diagnosis: Cholelithiasis. Choledocholithiasis. Obstructive jaundice. Cholangitis. Laparoscopic chole-
cystectomy (1.5 months ago)
2.what additional diagnostic methods
*MRCG:magnetic resonance cholangiography
*ERCP: Endoscopic cholangiopancreatography
*precutaneous transhepaatic cholangiography
* Investigations x Ultrasound abdomen—very useful, reveals presence or absence of gallstones; and thickening of gall bladder wall. Sonographic Murphy’s sign may be positive.
* Plain X-ray abdomen—10% of gallstones are radio-opaque; also rules out other causes of acute pain abdomen. Gas is seen in emphysematous GB.
* Total count shows neutrophilia.
* LFT is important. Increased serum bilirubin often signifies cholangitis or stone in the CBD.
* Plain X-ray abdomen is not very relevant but is often important to rule out duodenal ulcer perforation, peritonitis. Only 10% of gallstones are radio-opaque.
* CT scan is useful in identifying the perforation, impacted stone, gallbladder wall thickness and oedema
3. Make differential diagnosis
* acute pancreatitis
* malignant neoplasm of biliary tract,large duodenal papillary,metastasis to stomach,metastasis liver damage
* stricture and stenosis of large duodenal papilla (endemic cause opisthorchiasis
* Acute Cholecystitis
4. Determine patients further management tactics
*Endoscopic papillosphincterotomy or open surgical operation of choledocholitoextraction
* ERCP (retrograde cholangiopancreatography) with papillosphincterectomy and lithoextraction (removal of calculus) (the stone either falls into the duodenum lumen on its own, or with the Dormia basket). visualised. It is done under C-arm guidance and sedation like midazolam or propofol anaesthesia.
• In case of relapse in the postoperative period of the clinic of the disease, cholecystectomy, without waiting for the relief of jaundice.
• If there is no relapse, then planned cholecystectomy.
5.list possible complications in case of inadequate treatment
*liver failure
sepsis due to progression of cholangitis
* Due to the obstruction of the bile duct stone, the bile pressure increases above 270 mm of water column, the expansion and rupture of the bile capillaries, and the flow of bile into the blood or through the lymphatic vessels. The appearance of bile in the blood leads to direct hyperbilirubinemia (an increase in the content of conjugated bilirubin), hypercholesterolemia, the development of cholemic syndrome due to the circulation of bile acids in the blood, bilirubinuria (urine “beer color” and the presence of bile acids in the urine. Bile does not enter the intestine from for mechanical obstruction in the biliary tract leads to the fact that stercobilin is not formed and excreted in the feces (discolored, acholic feces).
#
**
Additional information

  1. The most likely diagnosis is common bile duct (CBD) stone or stricture, causing obstructive jaundice and post-cholecystectomy syndrome.
  2. Additional examination: Endoscopic retrograde cholangiopancreatography (ERCP) to visualize the CBD and diagnose any stones or strictures. The results may show CBD dilation and filling defects consistent with stones or narrowing of the CBD suggestive of a stricture.
  3. Differential diagnosis:
    - Postoperative complications such as bile leak or hematoma
    - Hepatitis or other liver diseases causing jaundice
    - Pancreatic cancer or other tumors causing biliary obstruction
    - Drug-induced liver injury or cholangitis.
  4. Management tactics:
    - ERCP with sphincterotomy and stone extraction or stent placement to relieve biliary obstruction
    - Antibiotics to treat any associated cholangitis
    - Monitoring of liver function tests and imaging studies to assess for resolution of the obstruction and any complications.
  5. Possible complications of inadequate treatment include:
    - Cholangitis or sepsis
    - Liver failure or cirrhosis
    - Recurrent biliary obstruction or stones
    - Pancreatitis or other complications of ERCP.
How well did you know this?
1
Not at all
2
3
4
5
Perfectly
2
Q

A young man, 26 years old, complains of pain in his right hand for a week. A corn appeared on the palm in the area of the pad between 3 and 4 fingers after physical work. The corn festered and appeared pronounced swelling of the back surface of the hand. The patient opened the corn abscess еindependently at home three days ago. A meager amount of pus was released from the abscess. But after a few hours the pain in the hand intensified, spread to the entire palm, the back hand swelling began to increase.
Objectively: the fingers of the right hand are evenly swollen, are in a half-bent position, slightly apart from each other, both flexion and extension of 3 and 4 fingers are painful. There is a wound 1x0.5 cm surface devoid of epidermis with a scanty discharge in the area of the interdigital space of 3 and 4 fingers. The palm fossa of the hand is smoothed, painful on palpation. There is soreness of the anterior surface of the distal part of the forearm, swelling on the dorslim of the hand. Probe palpation of the back of the hand, as well as along the flexor tendons at the level of the main and middle phalanges of all fingers is slightly painful and it is sharply painful in the projection of palmar aponeurosis. Body temperature is increased to 39 °C. Painful enlarged lymph nodes are palpated in the axillary region.
Task:
1. Formulate a diagnosis.
2. Justify the diagnosis
3. Describe the sequence of complications and the path of the spread of the purulent-inflammatory process in the analyzed clinical situation.
4. List what other complications are possible for this patient and why?
5. Where and what treatment is given to the patient?

A

1.Diagnosis: Primary diagnosis: Callus palm abscess of the right hand in area
in III-IV interdigital space.or Callus abscess of the palmar surface of the right hand in the region of the III-IV interdigital space, complicated by commissural phlegmon and phlegmon of the median palmar space and phlegmon of Pirogov-Paron space(fluid buildup 100-300ml )Axillary lymphadenitis.
2. Justify the diagnosis
*clinical features such as the fingers of the right hand are evenly swollen, are in a half-bent position, slightly apart from each other, both flexion and extension of 3 and 4 fingers are painful. There is a wound 1x0.5 cm surface devoid of epidermis with a scanty discharge in the area of the interdigital space of 3 and 4 fingers. The palm fossa of the hand is smoothed, painful on palpation. There is soreness of the anterior surface of the distal part of the forearm . Painful enlarged lymph nodes are palpated in the axillary region. And epidemiological data showing week. A corn appeared on the palm in the area of the pad between 3 and 4 fingers after physical work. The corn festered and appeared pronounced swelling of the back surface of the hand. The patient opened the corn abscess еindependently at home three days ago. A meager amount of pus was released from the abscess. But after a few hours the pain in the hand intensified, spread to the entire palm, the back hand swelling began to increase.
3. Describe the sequence of complications and path of the spread of the purulent inflammatory process in the analyzed clinical situation
* What causes osteomyelitis? Osteomyelitis occurs when bacteria from nearby infected tissue or an open wound circulate in your blood and settle in bone, where they multiply.
sepsis: Sepsis is the body’s extreme response to an infection. It is a life-threatening medical emergency. Sepsis happens when an infection you already have triggers a chain reaction throughout your body. Infections that lead to sepsis most often start in the lung, urinary tract, skin, or gastrointestinal tract,
* Myositis is a disease that makes your immune system attack your muscles. It causes chronic inflammation — swelling that comes and goes over a long time. Eventually, this inflammation makes your muscles feel increasingly weak. It can also cause muscle pain.
4.list what other complications are possible and why
* osteomyelitis
* myositis
* sepsis
5. Where and what treatment is given to the patient
* Drainage of wounds is carried out using PVC fenestrated tubes. After the operation, immobilization of the hand and forearm is required.
* Amoxicillin / clavulanate - 0.875-0.5 g 3 times / day; Cefotaxime 1.0-2.0 g 2 times / day; Ceftriaxone 1.0 g 2 times / day i / m, i / v; Erythromycin 0.5 g 4 times / day - taking into account the sensitivity of the microflora.
* Treatment should focus on:
Cleansing a purulent wound, healing without the formation of a rough scar, prevention of complications (osteomyelitis, myositis, sepsis).
**
Additional information

  1. Diagnosis: Acute purulent tenosynovitis and lymphangitis of the right hand with lymphadenitis of the axillary lymph nodes.
  2. Justification: The patient has a history of physical work and the appearance of a corn on the palm, which progressed to a festering abscess. The opening of the abscess did not relieve the symptoms and instead, the pain intensified, the hand swelling increased, and there is a wound with scanty discharge in the interdigital space. The patient also has a fever, painful enlarged lymph nodes in the axillary region, and probe palpation of the hand and forearm is painful. These symptoms are indicative of a bacterial infection that has spread to the tendons and synovial sheaths of the hand, causing tenosynovitis, and has also spread through the lymphatic vessels, causing lymphangitis and lymphadenitis.
  3. Sequence of complications and path of spread: The infection likely started as a superficial skin infection from the corn, which progressed to a deeper infection involving the tendons and synovial sheaths of the hand. The infection then spread through the lymphatic vessels, causing lymphangitis and lymphadenitis in the axillary region.
  4. Other possible complications: If left untreated, the infection can spread to other parts of the body, leading to sepsis and septic shock. It can also cause permanent damage to the tendons and joints of the hand, leading to decreased hand function and disability.
  5. Treatment: The patient needs urgent hospitalization and treatment with intravenous antibiotics, surgical drainage of the abscess and debridement of the infected tissue, and immobilization of the affected hand. Pain management and supportive care are also important.
How well did you know this?
1
Not at all
2
3
4
5
Perfectly
3
Q

A 30-year-old woman had pain in the epigastric region about 6 hours ago, a single vomiting of gastric contents. She was examined by a polyclinic doctor, her condition is satisfactory. The tongue is moist and slightly overlaid. The abdomen is not swollen, soft, slightly painful in the epigastrium and right iliac region. There are Positive Rovsing`s sign, Psoas sign, Obturator sign (Zachary Cope), localized abdominal tenderness, muscle guarding and limitation of respiratory movement in the lower abdomen. There are negative peritoneal symptoms. Body temperature 37.8℃, heart rate 80. Blood leukocytes 9,0x109/l. Gynecological anamnesis is without features, the next menstruation should begin in 2-3 days.
Task:
1. What is the most likely diagnosis?
2. Justify your diagnosis.
3. Specify the options for the location of the appendix and the features of the clinical picture of acute appendicitis, depending on the localization of the appendix.
4. Which urological, gynecological and infectious diseases most often have to differentiate appendicitis
5. What should be the algorithm of actions of a general practitioner in the analyzed clinical situation?

A
  1. Diagnosis: Acute appendicitis.(either phlegmonous or gangrenous appendicitis)
  2. Justify your diagnosis: Due to the following
    *pain in the right iliac region and epigastrium
    *vomiting of gastritis contents
    *positive symptoms of Rovsing,psoas sign, obturator sign( zachary cope), localized abdominal tenderness
    *Blood leukocytes 9.0 *10’9
  3. Specify the options for the location of the appendix and the features of the clinical picture of acute appendix
    *ascending appendix
    *medial appendix
    *ventral appendix
    *Retroceal ( in relation to peritoneum
    *pelvic
    ## ascending: the pain is localised in the right hypochondrium and can stimulate a clinic of biliary colic or peptic ulcer,more often than with typical forms are accompanied by vomiting due to irritation of duodenum.The location of the process near the extrahepatic bile ducts can cause transient jaundice
    #medial location:the process was shifted to the median line and was located close to the root or on the root of mesentery of the small intestine,which determined the clinical features of medial appendicitis.the appearance of pain syndrome from the very beginning is accompanied by repeated vomiting,which is associated with reflex irritation of the mesentry root.The pain was localised close to the navel
    #pelvic position:The inflamed process mayb come into contact with the bladder wall,which is manifested by dysuria and lower localised pain .
    #retroceal or pretriperitoneal position:the symptoms increase more slowly,which often leads to late hospitalisation more often there js irradiation in the right hip and even in the right hip joint
    ##The left -sided location of the vermiform process if extremely rare(0.1%) of observations.Most often with excessive mobility of the right half of the colon,when the process is either free or fixed in the right half of the colon,when the process is either free or fixed in the left half of the abdominal cavity.The clinical manifestations of the disease differ only localisation of the process,since all local signs of it are found in the left iliac region.Also the peculiarity of the pain localisation maybe be associated with pregnancy,especially in the second half,when the enlarging uterus shifts up and lateral the ileocecal angle,respectively,the pain will be localised in the right lateral area or in the right hypochondrium
  4. What urological, gynecological and infectious diseases most often have differentiate appendicitis
    Gynecological
    * Ectopic pregnancy
    *Rupture of ovarian cyst
    *ovarian apoplexy , ovarian torsion
    *pregnancy
    Infectious disease
    *salmonellosis
    *yersiniosis
    *pseudotuberculosis
  5. Algorithm of actions of a general practitioner
    * surgical operation should be done within 2 hours after establishment of diagnosis
    * laparoscopic appendectomy under general anesthesia
    *** Additional information
  6. The most likely diagnosis is acute appendicitis.
  7. Justification: The patient has pain in the epigastric region that has progressed to localized abdominal tenderness and muscle guarding in the lower abdomen. There are positive Rovsing’s sign, Psoas sign, and Obturator sign, which are indicative of irritation of the appendiceal peritoneum. The patient also has a slightly elevated body temperature and an increase in white blood cells, which are common findings in acute appendicitis.
  8. The appendix can have different locations in the abdomen, which can affect the clinical picture of acute appendicitis. If the appendix is retrocecal, the patient may experience pain in the right flank or back. If the appendix is pelvic, the patient may experience pain in the suprapubic region. If the appendix is subhepatic, the patient may experience pain in the right upper quadrant. However, regardless of the location, the patient will experience localized abdominal tenderness, muscle guarding, and other signs of peritoneal irritation.
  9. The general practitioner should differentiate appendicitis from urological conditions such as urinary tract infections and kidney stones, gynecological conditions such as ovarian cysts and ectopic pregnancy, and infectious diseases such as gastroenteritis and pneumonia.
  10. The algorithm of actions for a general practitioner in this clinical situation would be to refer the patient to a surgical specialist for further evaluation and possible surgical intervention. The general practitioner should also provide supportive care, including pain management and monitoring of vital signs.
How well did you know this?
1
Not at all
2
3
4
5
Perfectly
4
Q

An 82-year-old patient was taken to the surgical clinic by ambulance. Severe abdominal pain all over the abdomen appeared 6 hours before contacting the clinic. Also severe weakness, cold sticky sweat, bloating, there was double vomiting, which did not bring relief, loose stools with an admixture of dark purple blood in a small amount. On examination, the patient’s condition is severe. The tongue is dry. The abdomen is evenly swollen, palpation determines pronounced soreness in meso- and hypogastria, more on the right. The transfer pulsation of the abdominal aorta is preserved. Auscultative - peristaltic noises are poorly listened to. Peritoneal symptoms in the middle and lower floor of the abdominal cavity are doubtful.
History of coronary artery disease. Atrial fibrillation. Hypertension.
UAC : Hb 95 g/l, Er.2.81012/l, CP 1.0 L 19.6109/l, S/I 77%, Eos 1%, Lymph 16%, Mono 3%, ESR 16 mm/h.
OAM: yellow, sl mut., specific gravity 1015, protein 017., glucose rel., leukocyte 0-1 in p/s, epithelium ed in p/s.
Biochemical blood analysis: total/direct bilirubin 10.6/0 mmol/l, alpha-amylase 83 E/l, creatinine 0.12 mmol/l, glucose 6.9 mmol/l, urea 9.1 mmol/l, AST 14e/l, ALT 20 e/l, total protein 70 g/l.
SSC: fibrinogen total 3.73 g/l, INR 0.5 , RFMC 5.0 g/l
Overview radiography of the abdominal cavity: pronounced pneumatization of the loops of the small intestine with forming horizontal levels and Cloiber bowls, a single colonic level on the right is determined.
Ultrasound OBP: Conclusion: Hepatomegaly. Pronounced diffuse changes in liver and pancreatic tissue. GI with signs of chronic cholecystitis.
Task:
1. What is the most likely diagnosis?
2. Justify your diagnosis.
3. With what diseases is it necessary to carry out a differential diagnosis?
4. Against the background of which diseases does this pathology usually develop?
5. What treatment is indicated for this patient?

A

1:diagnosis: acute vascular insufficiency of the intestine (superior mesenteric artery thrombosis). Com-
plications: dynamic ileus,anemia moderate severity
intestinal necrosis? peritonitis?
2. Justification of diagnosis
*instrumental data given such as Overview radiography of the abdominal cavity: pronounced pneumatization of the loops of the small intestine with forming horizontal levels and Cloiber bowls, a single colonic level on the right is determined: history of causes of mesentery intestinal thrombosis such as coronary artery disease,atrial fibrillation and hypertension, Also some changes in the laboratory data given such as hb 95 which is decreased ,loose stools with an admixture of dark purple blood in a small amount
3 . Carry out differential diagnosis
* acute pancreatitis
* ruptured abdominal aortic aneurysm
* abdominal form of acute myocardial infarction
* mechanical obstruction of intestine
* gastrointestinal perforation
## Additional examinations
*CT scan
*Angiogram
*coagulation test, prothrombin time(pT), Activated pT
4.Against the background of which diseases does this pathology usually develop
*atherosclerosis
*thromboembolism
5. What treatment is indicated for this patient
*hospitalization
*in the absence of peritonitis (nasogastric intubation),prevention of compartment syndrome by decompression
*1st line surgical intervention: surgeons exercism away from the intestine that is healthy
*prescribe heparin 5000 iu 4 times a day in total 20,000
*Antibiotics
* Antispasmodic ( papaverine,dotovarine,
vitamin C
* patient monitoring
**

Additional information

  1. The most likely diagnosis is acute mesenteric ischemia (AMI).
  2. Justification: The patient’s symptoms of severe abdominal pain, vomiting, bloody stools, and signs of shock, along with the physical examination findings of diffuse abdominal tenderness and absent bowel sounds, are consistent with AMI. The presence of chronic cholecystitis on ultrasound suggests underlying vascular disease.
  3. Differential diagnosis: Other possible causes of acute abdominal pain and bloody stools include acute pancreatitis, acute diverticulitis, acute colitis, and gastrointestinal bleeding. These should be ruled out with further diagnostic tests.
  4. Diseases associated with AMI: AMI usually develops against the background of underlying vascular disease, such as atherosclerosis, thromboembolism, or mesenteric venous thrombosis. The patient’s history of coronary artery disease and atrial fibrillation may increase their risk for vascular events.
  5. Treatment: The patient requires urgent surgical intervention, such as thrombectomy or revascularization, to restore blood flow to the affected mesenteric vessels. Antibiotics may be given to prevent infection. Supportive care, such as fluid resuscitation and electrolyte management, should also be provided.

***Additional information

  1. The most likely diagnosis is acute mesenteric ischemia.
  2. Justification: The patient has severe abdominal pain all over the abdomen, which is a common symptom of acute mesenteric ischemia. The patient also has weakness, cold sticky sweat, bloating, double vomiting, and loose stools with an admixture of dark purple blood, which are indicative of intestinal ischemia and necrosis. The abdominal examination reveals pronounced soreness in the meso- and hypogastria, and the peristaltic noises are poorly listened to, which are also indicative of mesenteric ischemia.
  3. The differential diagnosis should include acute pancreatitis, acute cholecystitis, peptic ulcer disease, and aortic aneurysm.
  4. This pathology usually develops against the background of atherosclerosis, thromboembolism, or arterial embolism, which can cause occlusion of the mesenteric arteries and lead to intestinal ischemia and necrosis.
  5. The treatment for this patient should be urgent hospitalization and surgical intervention, including revascularization of the mesenteric arteries and resection of the necrotic bowel. Supportive care, including fluid and electrolyte management, pain management, and monitoring of vital signs, is also important. The patient may also require antibiotics to prevent infection.
How well did you know this?
1
Not at all
2
3
4
5
Perfectly
5
Q

A 45-year-old woman, who has not been ill before and has not been examined, complains of pain in the right hypochondrium with irradiation into the right half of the chest, nausea, repeated vomiting of gastric contents with an admixture of bile. She became acutely ill 2 days ago - around 8 o’clock in the evening, there were quite severe pains described above, the intensity of which decreased the next morning.
is observed for two days. The general condition is satisfactory. The subjectivity of the sclera is noted. Ortner’s symptom is positive(pain in touching the costal margin )a painful infiltrate with indistinct contours is palpated in the right hypochondrium. There are no signs of peritoneum. Urine diastasis is 16 units.
Task:
1. What is the most likely diagnosis? Justify your diagnosis.
2. Make a protocol of examination of the patient.
3. What are your treatment tactics?
4. Indications for surgical treatment of this pathology?
5. Indicate the probable causes of this disease.

A

1 . Diagnosis: Cholelithiasis. Acute phlegmonous calculous cholecystitis.Acute inflammatory infiltrate .bstructive jaundice.
Justification:pain syndrome , vomiting of gastric contents with an admixture of bile. She became acutely ill 2 days ago, Subfertility (phlegm-on cholecystitis) is observed for two days. The general condition is satisfactory. The subjectivity of the sclera is noted. Ortner’s symptom is positive(pain in touching the costal margin )a 🏮painful infiltrates with indistinct contours is palpated in the right hypochondrium.
2.make a protocol of examination
laboratory diagnosis * increases leucocytosis >10,00010’9, left shift, increased ESR/mm per hour, c.reactive proteins (++++)
* Investigations x Ultrasound abdomen—very useful, reveals presence or absence of gallstones; and thickening of gall bladder wall. Sonographic Murphy’s sign may be positive.
* Plain X-ray abdomen—10% of gallstones are radio-opaque; also rules out other causes of acute pain abdomen. Gas is seen in emphysematous GB.
* Total count shows neutrophilia.
* LFT is important. Increased serum bilirubin often signifies cholangitis or stone in the CBD.
* Plain X-ray abdomen is not very relevant but is often important to rule out duodenal ulcer perforation, peritonitis. Only 10% of gallstones are radio-opaque.
* CT scan is useful in identifying the perforation, impacted stone, gallbladder wall thickness and oedema
*ERCP
*MRCP
3, what are your treatment tactics
* Advised hospitalisation.
*Initially (nonoperative) conservative treatment (95%):
*Nasogastric aspiration. IV fluids. *Analgesics and antispasmodics(papaverine 20mg/
*Broad spectrum antibiotics (cefoperazone, ceftazidime, ceftria xone, cefotaxime + amikacin, tobramycin + metronidazole {antimicrobial}).
*Observation.
*Follow-up U/S scan.
* Later after 3–6 weeks, elective interval cholecystectomy, either by open method through right subcostal (Kocher’s) incision or through laparoscopy is done.
4. Indications for surgical treatment of this pathology
* Empyema GB
*Persisting symptoms/failure of medication
*Emphysematous cholecystitis *Perforation/peritonitis
*Elderly patients
##surgery should be performed within 72 hours (3 days) at the onset of the disease Here the gallbladder is opened and all stones and pus are removed. Either a Foley’s or Malecot’s catheter is placed in the gallbladder and is exterio rised. US-guided percutaneous cholecystostomy also can be used. After 3 weeks, elective cholecystectomy is done. Do not feed the patient orally during the surgical stage of treatment
5. Indicate the probable causes of this disease
* Usual cause is impacted gallstone in the Hartmann’s pouch, obstructing cystic duct. Pathogenesis; Stone causes obstruction at Hartmann’s pouch or in cystic duct. Obstruction causes stasis, oedema of the wall, bacterial infection, acute cholecystitis and its effects.
*acute opisthochiasis
*strictures
**
Additional information

  1. The most likely diagnosis is acute cholecystitis, based on the patient’s symptoms of right upper quadrant pain, nausea, vomiting, fever, and positive Ortner’s sign (pain on tapping the right costal margin). The tender infiltrate palpated in the right hypochondrium suggests inflammation of the gallbladder.
  2. Examination plan:
    - Blood tests to assess liver function, white blood cell count, and inflammatory markers
    - Ultrasound of the abdomen to confirm the diagnosis and assess for complications such as gallbladder distension or stones
    - CT scan or MRI if there is suspicion of complications such as abscess or perforation
    - HIDA scan to evaluate gallbladder function if the ultrasound is inconclusive
  3. Treatment strategy:
    - NPO (nothing by mouth) status to rest the gallbladder and prevent further inflammation
    - Pain management with analgesics such as NSAIDs or opioids
    - Antibiotics to cover common pathogens causing acute cholecystitis
    - Cholecystectomy (surgical removal of the gallbladder) is the definitive treatment and is usually performed after the acute episode resolves.
  4. Indications for surgery:
    - Acute cholecystitis with persistent symptoms despite medical management
    - Complicated cholecystitis such as gallbladder perforation, abscess, or empyema
    - Recurrent episodes of acute cholecystitis
    - Presence of gallstones or polyps in the gallbladder
  5. Possible causes of acute cholecystitis include:
    - Gallstones blocking the cystic duct
    - Infection of the gallbladder
    - Ischemia or reduced blood flow to the gallbladder
    - Tumors or polyps in the gallbladder
    - Trauma to the gallbladder or biliary system.
How well did you know this?
1
Not at all
2
3
4
5
Perfectly
6
Q

The patient, 38 years old, complained of epigastric pain, increasing after eating, weight loss, periodic increases body temperature to subfebrile numbers. About a year ago, he was treated in a hospital for acute pancreatitis. Abusing alcohol. He noted worsening in the condition during the last 6 months. Underwent 3 distinct attacks on an outpatient basis. At the time of examination, the condition is relatively satisfactory. The abdomen participates in breathing, soft, almost painless. There is a scar from laparocentesis near the navel. An infiltrate is palpated in the epigastrium, a structure without clear contours, stationary. Blood count test and urine, bilirubin, urea, creatinine, amylase, alkaline phosphatase, transaminases (ALT, AST) - within normal limits. Ultrasound - a fluidic structure up to 8 cm with well-defined walls is determined in the area of the body and tail of the pancreas, without internal structures.
Task:
1. What is the most likely diagnosis?
2. List the necessary additional examinations.
3. What are your treatment tactics?
4. Specify the possible causes of this disease.
5. Types of surgical treatment for this pathology.

A
  1. Diagnosis: Recurrent chronic toxic pancreatitis (alcoholic). Postnecrotic pseudocyst of the pancreas.
    2.Additional examinations
    *nutriotional studies -patient BMI, albumin,total protein ,transferrin ,magnesium ,zinc
    *MRI
    *Endosonography possibly to do a tissue biopsy
    * Ultrasound - controlled antegrade cysto- and pancreaticography (virungography).
    * US abdomen (commonly done procedure), US reveals the size and thickness of the pseudocyst. Size less than 6 cm indicates that one can wait for spontaneous resolution. *Endosonography (EUS) is very useful. * CT scan is ideal and study of choice. It is two times more sensitive than US. It demonstrates size, shape, number, wall thickness, contents, pancreatic duct size, and extent of necrosis in pancreas, calcification and atrophy in chronic pancreatitis, regional vessels, pseudoaneurysm, splenic/ portal vein thrombosis.
    *MRCP delineates the ductal anatomy and its abnormality.
    *ERCP is done to find out the communication.
    *Barium meal (lateral view) shows widened vertebrogastric angle with displaced stomach (Not usually done now).
    *LFT, serum amylase platelete count, PT INR.
    *EUS-guided aspiration and analysis of fluid for amylase and CEA. Amylase will be high with normal CEA in pseudocyst; amylase will be normal/low with high CEA >400 ng/ml in mucinous neoplasm.
  2. Specify the possible causes of this disease
    * It is localized collection of sequestered pancreatic fluid, usually 3 weeks after an attack of acute pancreatitis.
    *It can occur after trauma and recurrent chronic pan creatitis. *Collection usually occurs in the lesser sac in rela tion to stomach, but can occur in relation with duodenum, jejunum, colon, splenic hilum.
    *About 50% of acute pancreatitis leads to pseudo cyst formation, but among that 20–40% will resolve spontaneously
  3. Types of surgical treatment for this pathology
    *resection ,imposition of cytodigestive anastomosis ,stent drainage
    ## Drainage operations for postnecrotic pancreatic cysts
    Indications for drainage of postnecrotic cysts are the size of the cyst more than 5 cm, abscess formation, pain, impaired outflow of bile or food from the stomach, portal hypertension.
    * Endoscopic drainage of the cyst cavity into the duodenum or stomach . Endoscopic drainage is performed under endosonographic control in cases where the cyst is adjacent to the wall of the duodenum (preferably) or stomach, in the absence of large sequesters in the cyst cavity. Self-expanding coated stents should be used for drainage. Stents should not be removed until instrumental confirmation of the collapse of the cyst walls and earlier than 2 months after their implantation.
    * Cystopancreatojejunostomy Minimally invasive approach according
    to M.I. Prudkov should be preferred. Anastomosis of the cyst should be performed with a loop of the jejunum disconnected according to Roux, at least 60 cm long, with a single-row twisted suture with a monofilament absorbable suture. The jejunal ru-loop is placed in front of the colon. An end-to-side interintestinal anastomosis is applied at the level of the first vascular arcade of the mesentery of the jejunum. The walls of the cyst are not excised, with the exception of a small fragment, which is strictly necessarily subjected to histological examination.
    * External drainage of postnecrotic cysts is a forced palliative measure and is performed in case of abscess formation or an extremely serious condition of the patient.

*** Additional information

  1. The most likely diagnosis is chronic pancreatitis with a pseudocyst formation. The patient has a history of acute pancreatitis and alcohol abuse, which are common risk factors for chronic pancreatitis. The presence of an infiltrate in the epigastrium and a fluidic structure in the area of the body and tail of the pancreas on ultrasound suggest a pseudocyst formation.
  2. Additional examinations:
    - CT scan or MRI to confirm the diagnosis and assess the extent of the disease
    - Endoscopic retrograde cholangiopancreatography (ERCP) or magnetic resonance cholangiopancreatography (MRCP) to evaluate the pancreatic and biliary ducts
    - Endoscopic ultrasound (EUS) to obtain a more detailed image of the pancreas and pseudocyst
  3. Treatment tactics:
    - Pain management with analgesics
    - Nutritional support with a low-fat diet and pancreatic enzyme supplements
    - Abstinence from alcohol
    - Endoscopic or surgical drainage of the pseudocyst if it is causing symptoms or complications such as infection, rupture, or obstruction of adjacent structures.
  4. Possible causes of chronic pancreatitis:
    - Alcohol abuse
    - Gallstones
    - Autoimmune diseases
    - Genetic mutations
    - Idiopathic (unknown cause)
  5. Types of surgical treatment for chronic pancreatitis with a pseudocyst:
    - Cystogastrostomy: creating an opening between the pseudocyst and the stomach to drain the fluid into the gastrointestinal tract
    - Cystojejunostomy: creating an opening between the pseudocyst and the jejunum to bypass the stomach
    - Roux-en-Y cystojejunostomy: a more complex procedure that involves rerouting the small intestine to create a new pathway for the pancreatic juices to reach the intestine and bypass the pseudocyst.
How well did you know this?
1
Not at all
2
3
4
5
Perfectly
7
Q

А 30-year-old patient had dysphagia a month ago. X-ray examination revealed that the volume of the left half of the chest was reduced due to fibrous adhesions. Centers of blackout are detected in both lungs. The esophagus is moderately dilated throughout, in a fasted state contains a significant amount of content. On the X-ray the contours of the esophagus are smooth, clear, the gas bubble of the stomach is not pronounced. Periodically, portions of barium penetrate into the stomach.
Task:
1. What is the most likely diagnosis? Justify your diagnosis.
2. Make a plan for the necessary examination of the patient.
3. Perform a differential diagnosis in this patient.
4. What is your therapeutic tactics?
5. Types of surgical treatment for this pathology.

A

1.Diagnosis: Achalasia of cardia, type II.
2.research plan for patient (examination)
* Barium swallow X-ray showing features of achalasia cardia;
* Chest X-ray shows patches of pneumonia. Double mediastinal strip of dilated oesophagus is typical with air fluid level in posterior mediastinum on lateral view.
* Oesophageal manometry shows unrelaxed lower oesophageal sphincter with high resting pressure—very useful and gold standard. It shows failure of LES to relax completely during swallowing and complete absence of peristalsis.
* Oesophagoscopy is done to confirm the diagnosis and to rule out carcinoma oesophagus. It shows totally closed LES (rosette like) with atonic, dilated proximal oesophagus. Biopsy of mucosa at LES should be done.
3. Perform a differential diagnosis in this patient
* Carcinoma oesophagus
* Stricture oesophagus.
* Scleroderma.
*Chagas’ disease
4:what is your therapeutic tactics
*Hospitalization of patient
* Intravenous fluids
*medication such Drugs: Botulinum toxin : Botulinum toxin is neurotoxic protein derived from Clostridium botulinum is highly toxic poisonous substance. Very small dose is used for therapeutic purpose. Seven types of toxins are found. A (A1 /A2 /A3) type is used for therapy. It blocks the cholinergic nerve ends reducing the cholinergic acetylcholine release causing flaccid paralysis of muscles. It is used in cosmetic facial line, strabismus, focal dystonia, tremor, tics, muscle spasms, achalasia, smooth muscle hyperactivity, Frey syndrome, and hyperhidrosis. , Nitroglycerin,nifedipine can be prescribed as a temporary treatment
* Then prepare patient for surgical treatment
5. Types of surgical treatments
*pneumatic dilation is the most effective way of reducing dysphagia in achalasia if there is contraindications for surgery
*drug therapy:Nifedipine,nitrosorbide )injections of botulism
* Surgery: Modified Heller’s cardiomyotomy with Toupet’s / Dor’s fundoplication : Modified Heller’s operation (Heller—German, 1913): Oesophagocardiomyotomy. Success rate is 85%. Heller’s cardiomyotomy for achalasia cardia. Only circular muscle layer is cut longitudinally in OG junction until mucosa protrudes out without perforating the mucosa; (B) Heller’s cardiomyotomy—on table look with partial fundoplication (Either through thoracic or through abdominal approach, thickened circular muscle fibres are cut longitudinally for about 8–10 cm, 2 cm proximal to the thickened muscle to 1 cm distal to OG junction. Care should be taken not to open the mucosa (anterior myotomy is done now. Original Heller’s is both anterior and posterior myotomies) Nissen’s or Toupet’s fundoplication is done along with the above procedure to prevent reflux. Resection is done when failure of myotomy occurs or when megaoesophagus or metaplasia is present. Transhiatal total oesophagectomy with gastric pull up and oesophagogastric anastomosis in the neck is a good option in such patients.
–Open abdominal/thoracic (rare) approach
-Laparoscopic approach—
* Plummer’s pneumotic dilatation Negus hydrostatic balloon dilatation
**
additional information

  1. The most likely diagnosis is achalasia. The patient presents with dysphagia, esophageal dilation, and a significant amount of content in the esophagus in a fasted state. The smooth and clear contours of the esophagus on X-ray, along with the periodic penetration of barium into the stomach, suggest a functional obstruction at the lower esophageal sphincter (LES), which is a characteristic feature of achalasia. The fibrous adhesions and centers of blackout in both lungs may be due to chronic aspiration secondary to the dysphagia.
  2. Plan for necessary examination:
    - Upper endoscopy to evaluate the esophageal mucosa and rule out other causes of dysphagia such as tumors or strictures
    - Manometry to confirm the diagnosis of achalasia and assess the LES function
    - CT scan or MRI to evaluate the extent of the fibrous adhesions and rule out other causes of lung pathology
  3. Differential diagnosis:
    - Esophageal stricture
    - Esophageal cancer
    - Gastroesophageal reflux disease (GERD)
    - Eosinophilic esophagitis
    - Neurological disorders affecting the esophagus
  4. Therapeutic tactics:
    - Pharmacological treatment with calcium channel blockers or nitrates to relax the LES
    - Botulinum toxin injection into the LES to temporarily paralyze the muscle
    - Endoscopic balloon dilation to stretch the LES
    - Surgical myotomy to cut the muscle fibers of the LES and improve esophageal emptying
  5. Types of surgical treatment for achalasia:
    - Laparoscopic Heller myotomy: a minimally invasive procedure that involves cutting the muscle fibers of the LES through small incisions in the abdomen
How well did you know this?
1
Not at all
2
3
4
5
Perfectly
8
Q

A 25-year-old patient, three years after the chest injury (left), with a sharp lifting of a heavy object, sharp pains appeared behind the sternum and in the left half of the chest with irradiation into the left shoulder, shortness of breath. Objectively: acrocyanosis, respiratory rate 40, tachycardia 120 per minute, atrial fibrillation. During auscultation, there are peristaltic noises in the left half of the chest.
Task:
1. What is the most likely diagnosis?
2. Name the necessary additional examinations.
3. What are your treatment tactics?
4. Name the radiological signs of this disease.
5. Surgical treatment options for this pathology.

A

1.Diagnosis: Diaphragmatic rupture. Incarcerated post-traumatic diaphragmatic hernia.
2.Additional examination research
*chest x-ray
*CT scan
*ultrasound
3. Treatment strategies
*Fast track surgery (laparostomy): Take the intestine grip ,respect the necrotic tissue and do anastomosis and suture the diaphragm with a mesh
4: Name the radiological signs of this disease
*displacement of borders of the heart in the opposite position,high standing and paradoxical movement of the diaphragm dome
*presence of air formations or horizontal levels in the chest cavity
5. Surgical treatment options for this pathology
*laparotomy,removal of the affected abdominal organs from the thoracic cavity
-Anatomoses
-suture of diaphragm
-Drain abdominal cavity
-closure of open cavity
*patient should be in a resuscitation department
* conservative treatment
-Antibiotics: metronidazole 500 mg , ceftriaxone
-intravenous fluids
-painkillers ( tramadol)
-Antispasmodics(Akineton, Artane, Bentyl, buscopan
**
Additional information

  1. The most likely diagnosis is spontaneous pneumothorax.
  2. Additional examinations:
    - Chest X-ray to confirm the diagnosis and assess the size and location of the pneumothorax
    - Arterial blood gas analysis to evaluate the patient’s oxygenation status and acid-base balance
    - CT scan or ultrasound to assess for any underlying lung disease or complications such as pleural effusion or hemothorax.
  3. Treatment tactics:
    - Immediate needle decompression or chest tube insertion to evacuate the air from the pleural space and re-expand the lung
    - Oxygen therapy to improve oxygenation
    - Pain management with analgesics
    - Monitoring for any complications such as tension pneumothorax or pleural effusion.
  4. Radiological signs of spontaneous pneumothorax include:
    - Absence of lung markings at the site of the pneumothorax
    - Shift of the mediastinum to the contralateral side
    - Visible pleural line and pleural effusion if present.
  5. Surgical treatment options for spontaneous pneumothorax include:
    - Video-assisted thoracoscopic surgery (VATS) with pleurodesis to prevent recurrence
    - Open thoracotomy with pleurectomy or bullectomy in cases of recurrent or persistent pneumothorax.
    *** Additional information
    1. The most likely diagnosis is diaphragmatic hernia. The patient has a history of chest injury, and lifting a heavy object may have caused a sudden increase in intra-abdominal pressure, leading to the herniation of abdominal contents through a defect in the diaphragm. The sharp pains behind the sternum and in the left half of the chest with irradiation into the left shoulder, along with the respiratory distress and peristaltic noises on auscultation, suggest the involvement of the gastrointestinal tract in the hernia. The acrocyanosis, tachycardia, and atrial fibrillation may be due to the compression of the heart and lungs by the herniated contents.
  6. Necessary additional examinations:
    - Chest X-ray to confirm the diagnosis and assess the extent of the hernia
    - CT scan or MRI to evaluate the size and location of the hernia and the herniated contents
    - Upper gastrointestinal endoscopy to assess the involvement of the gastrointestinal tract in the hernia
  7. Treatment tactics:
    - Stabilize the patient’s condition with oxygen therapy, fluid resuscitation, and medications to control the heart rate and rhythm
    - Surgical repair of the diaphragmatic hernia to reduce the herniated contents and close the defect in the diaphragm
    - In some cases, a temporary measure such as a nasogastric tube may be needed to decompress the herniated stomach and prevent further complications.
  8. Radiological signs of diaphragmatic hernia:
    - Abnormal location of abdominal organs in the chest cavity
    - Defect in the diaphragm
    - Compression of the heart and lungs by the herniated contents
    - Peristaltic noises on auscultation of the chest
  9. Surgical treatment options for diaphragmatic hernia:
    - Laparoscopic repair: a minimally invasive procedure that involves making small incisions in the abdomen and using a laparoscope to repair the hernia
    - Open repair: a traditional surgical approach that involves making a larger incision in the abdomen or chest to repair the hernia
    - Thoracoscopic repair: a minimally invasive procedure that involves making small incisions in the chest and using a thoracoscope to repair the hernia from the inside of the chest cavity.

Roemheld syndrome

Hernia duplication or
Suture of defect/ mesh for reinforcement or
Suture + diaphragm some plasty

How well did you know this?
1
Not at all
2
3
4
5
Perfectly
9
Q

53-year-old man has noted moderate dysphagia for the last six months, manifested by a feeling of “touching”, “scratching” when passing hard food through the esophagus. Stomach resection was 10 years ago. Hoarseness of voice appeared a week before hospitalization. With laryngoscopy: there is a decrease in the tone of the vocal fold. X-ray examination of the esophagus: at the level of ThVIII-IX determines the narrowing of the esophagus, a lumen of up to 5 mm with uneven contours. The gas bubble of the stomach is preserved.
Task:
1. What is the most likely diagnosis?
2. Name the necessary additional examinations.
3. What are your treatment tactics?
4. Name the signs of inoperable in this pathology.
5. Surgical treatment options for this pathology.

A
  1. Diagnosis: Lower esophagus cancer. Dysphagia, type 1
    Justification: patient has dysphagia and also in the radiological examination we can see 5mm with uneven contours also patient feels touching,scratching when passing hard food through the office
  2. Additional examination research
    * EGDS with biopsy (allows you to see the CO defect, determine its size and nature, take a tissue biopsy for histological examination)
    * X-ray examination of the esophagus with contrast (allows to determine the prevalence and extent of malignant neoplasm of the esophagus, to determine the tactics of surgical intervention)
    * Endo-ultrasound (ultrasonography) (assesses the depth of tumor invasion into the esophageal wall, assesses the state of regional l / u), puncture biopsy of mediastinal l / u is possible to determine staging and treatment tactics.
    * Clinical analysis and biochemical blood tests, tumor markers CEA (cancer-embryonic antigen-tissue marker of oncological diseases), CA 19-9 (tumor marker), study of the coagulation system, OAM.
    3.Treatment strategies
    * The “gold standard” for the treatment of cervical esophageal cancer is combined treatment - a combination of neoadjuvant chemoradiation therapy followed by resection of the lesion in a single block, lymphadenectomy and the formation of a reliable functional anastomosis. It is the operative stage of treatment that remains a difficult surgical task.
    Gastrostomy
  3. Signs of inoperable in this pathology
    * Stage IV esophageal cancer has spread to distant lymph nodes or to other distant organs. In general, these cancers are very hard to get rid of completely, so surgery to try to cure the cancer is usually not a good option.
  4. Surgical treatment options for this pathology
    * Esophagectomy is the most common form of surgery for esophageal cancer. During the procedure, surgeons: Remove all or part of your esophagus and nearby lymph nodes through incisions in your chest, abdomen or both. Reconstruct the esophagus using the stomach or colon.
    -Transhiatal Esophagectomy

Transhiatal esophagectomy is the most common type of esophageal cancer surgery we perform. This approach involves only a neck incision.

During a transhiatal esophagectomy, we:

Remove most of your esophagus and a little bit of your stomach
Bring the rest of your stomach up to your neck and attach it to the remaining esophagus
-Ivor Lewis Esophagectomy

Ivor Lewis esophagectomies are the second most common type of esophagectomy we perform. This approach involves incisions in the abdomen and chest. During this procedure, we:

Remove part of your esophagus
Make a tube (conduit) out of part of the stomach
Connect the stomach conduit to the esophagus in your chest
-Thoracoabdominal Gastrectomy

The thoracoabdominal approach is the least common. We use it when a tumor is in your lower esophagus or gastroesophageal (GE) junction (where your esophagus joins your stomach).

During this procedure, we:

Make an incision in your abdomen that goes to the left side of your chest
Divide your esophagus in the left chest
Bring either your stomach or small intestine up to your left chest to create a new connection

How well did you know this?
1
Not at all
2
3
4
5
Perfectly
10
Q

A 58-year-old man was taken to the emergency department with copious vomiting of “coffee grounds” and repeated melena, following the collapse 3 hours ago. For two weeks, this was preceded by general weakness, repeated black poorly designed stool. Over the past few years, he has noted pain in the epigastric region, which recently began to radiate into the back. I have never been treated or examined before.
The patient’s condition is of moderate severity, pulse 96 per minute, blood pressure 105/75 mm Hg.
FGDS: during urgent gastroscopy, a deep ulcerative crater 3.0 x 3.0 cm with active bleeding from a large vessel in the bottom of the ulcer was found in the area of the angle of the stomach.
Task:
1. Formulate your diagnosis?
2. What are your treatment tactics?
3. Prescribe conservative therapy to the patient.
4. Indications for emergency surgery in this patient?
5. Possible options for surgical treatment.

A
  1. Diagnosis: Peptic ulcer disease. Chronic ulcer of the esophagogastric angle (3x3 cm). Continuous
    bleeding.
  2. Treatment tactics
    * Urgent hospitalization. Preparation for surgical (endoscopic) bleeding control. Patients with complicated forms of gastric ulcer and duodenal ulcer are recommended to be hospitalized for the purpose of surgical treatment in a surgical hospital. It is advisable to start treatment of ulcer bleeding in the intensive care unit, as well as treatment for decompensation of chronic diseases in combination with other complications of peptic ulcer disease. The main task of treatment in the intensive care unit is to stabilize the patient’s condition - replenishment of the circulating blood volume, fight against manifestations of hypovolemic shock, adequate pain relief, treatment of decompensated concomitant diseases and systemic inflammatory response syndrome, after which surgical and / or endoscopic methods of treatment can be applied. Patients with bleeding ulcers are recommended to undergo esophagogastroduodenoscopy in order to verify the source of bleeding and perform endoscopic hemostasis in the first 2 hours after hospitalization.
    -endoscopic hemostasis
    * Endoscopic hemostasis methods:
    • injection (exposure to hemostatic and vasoconstrictor agents, such as adrenaline solution);
    • diathermocoagulation (mono- and bipolar);
    • laser coagulation;
    • argon-plasma coagulation;
    • clipping (including “tri-clip”);
    • use of sclerosing compositions;
    3a. Additional examination methods
    * if endoscopic is ineffective - surgical hemostasis as early as possible
    Laboratory: KLA, coagulation system, OAM, HD (ALT, AST, protein, creatinine, glucose.), Chest x-ray.
    3b. Prescription of conservative treatment
    * The use of PPIs contributes to stopping ulcerative bleeding and reducing the risk of re-bleeding. At the same time, 80 mg of esomeprazole is injected simultaneously bolus intravenously, followed by continuous intravenous infusion of this drug (at a dose of 8 mg per hour) for 72 hours. After the patient is transferred to oral administration of drugs, eradication therapy is performed.
    * Replenishment of blood loss, stabilization of hemodynamics: -solutions of crystalloids 30-40 mg / kg, colloids (albumin)
    - With a deficiency of blood clotting factors, transfusion of fresh frozen plasma is indicated.
    * In case of hypoxia, oxygen therapy is required.
    * Vasopressors are indicated in case of insufficient effectiveness of infusion-transfusion therapy.
    * Subsequently - Consultation of a physician, research on HP, eradication therapy according to indications: omeprazole 20 mg x 2 times a day, amoxicillin 1000 mg 2 times a day and clarithromycin 500 mg 2 times a day for 10-14 days.
  3. Indications for emergency surgery in this patient
    * Emergency surgery is indicated in patients with ongoing bleeding with ineffectiveness (or impossibility) of endoscopic hemostasis or with recurrent bleeding;
    5.possible options for surgical treatment
    * In patients at high risk of surgery, it is preferable to perform X-ray endovascular selective angiography followed by occlusion of the bleeding vessel.
    * Gastrotomy with suturing of a bleeding ulcer and suturing of the defect.
    * For bleeding stomach ulcers, it is recommended: stem vagotomy (consists in crossing the vagus nerve trunks and crossing the small branches along the entire circumference of the esophagus for at least 6 cm above the hepatic and celiac branches leaving them) with economical gastrectomy and gastrojejunal anastomosis according to Roux or Billroth- I. Russian scientific surgical community recommends this tactic
    ** additional information
  4. The diagnosis is a bleeding gastric ulcer. The patient presents with copious vomiting of “coffee grounds” and repeated melena, along with a history of epigastric pain radiating to the back and black poorly formed stool. The deep ulcerative crater with active bleeding from a large vessel in the bottom of the ulcer found during urgent gastroscopy confirms the diagnosis.
  5. Treatment tactics:
    - Stabilize the patient’s condition with intravenous fluids, blood transfusions, and medications to control bleeding and acid secretion
    - Endoscopic therapy with injection of epinephrine or application of hemostatic clips to stop the bleeding
    - Proton pump inhibitors to reduce acid secretion and promote healing of the ulcer
    - Antibiotics to treat Helicobacter pylori infection if present
    - Avoidance of nonsteroidal anti-inflammatory drugs (NSAIDs) and other irritants to the gastric mucosa.
  6. Conservative therapy for the patient:
    - Intravenous fluids and blood transfusions to replace lost fluids and blood
    - Proton pump inhibitors to reduce acid secretion and promote healing of the ulcer
    - Antibiotics to prevent infection if there is evidence of H. pylori infection
    - Avoidance of food and drink until the bleeding stops and the patient’s condition stabilizes.
  7. Indications for emergency surgery:
    - Failure of endoscopic therapy to stop the bleeding
    - Hemodynamic instability despite resuscitation
    - Recurrent bleeding after initial successful endoscopic therapy
    - Perforation or penetration of the ulcer
    - Obstruction of the gastric outlet due to scarring or edema
  8. Possible options for surgical treatment:
    - Partial gastrectomy: removal of the affected part of the stomach
    - Vagotomy: cutting of the vagus nerve to reduce acid secretion and promote healing
    - Pyloroplasty: widening of the pyloric sphincter to improve gastric emptying
    - Hemigastrectomy: removal of one half of the stomach
    - Gastrojejunostomy: creation of a new opening between the stomach and the jejunum to bypass the ulcer.
How well did you know this?
1
Not at all
2
3
4
5
Perfectly
11
Q

A 40-year-old woman complains of cramping abdominal pain, repeated copious vomiting with an admixture of bile. Ill for three days. A few years ago, she underwent a cholecystectomy for chronic calculous cholecystitis. Objectively: the abdomen is asymmetrical and slightly painful, without peritoneal symptoms, the “splash noise” is determined. With an overview X-ray of the abdomen, multiple small intestine levels are determined.
Task:
1. What is the most likely diagnosis?
2. List the necessary additional examinations.
3. What are your treatment tactics?
4. The basic principles of conservative therapy?
5. Options for surgical treatment for this pathology.

A

1.Diagnosis: Peritoneal adhesions of the intestine. Acute intestinal obstruction.
2.list necessary additional examinations
* X-ray of the OBP with contrast or Computed tomography (CT) with double (oral and intravenous) contrast,
* ultrasound of the OBP - violations of blood supply in the intestinal wall are: the presence of an expanded “isolated loop” of the small intestine with fluid, thickening, heterogeneity of the wall in combination with its akinesia , and accumulation of free fluid in the abdominal cavity
* general blood analysis
* general urine analysis
* blood test for acid base balance
* study of blood electrolytes, blood sugar
* blood group, Rh factor.

The presence of leukocytosis more than 14x109 L, the appearance of acidosis, amylasemia most likely indicates the presence of strangulation.
3. What are your treatment tactics
* Aspirate contents from stomach and intestines
* Catheterization of a peripheral vein, restoration of the BCC, elimination of water-electrolyte disturbances and measurement of central venous pressure (monitoring the usefulness of infusion therapy).
* Bladder catheterization. After preparation, surgical intervention is performed depending on the specified diagnosis
4. The basic principles of conservative therapy
*Hunger . Rest Bed rest,
*Infusion therapy (40ml/kg,crystalloids),correction of blood electrolytes, antispasmodic
* gastric lavage, aspiration of duodenal and intestinal contents, siphon enemas, antispasmodic or anticholinesterase drugs) in the absence of a pronounced effect should be carried out no more than 2 hours.
* Control X-ray examination of the abdominal organs allows to determine the results of conservative treatment of acute intestinal obstruction.
* An absolute contraindication to conservative therapy is the growing signs of the syndrome of a systemic reaction to inflammation, an increase in the degree of dysfunction of the cardiovascular, respiratory system, kidneys, increased intoxication, and the appearance of signs of peritonitis.
5. Options for surgical treatment for this pathology
*upper middle laparotomy,revision of the abdominal cavity,adesioslysis,installation of a nasoinstestinal probe,drainage of the abdominal cavity.it is necessary to examine the abdominal cavity during surgery,to identify the level of intestinal obstruction,to release the pinched loop of the intestine
* . In acute adhesive small bowel obstruction, it is recommended to perform a minimally invasive intervention - laparoscopic adhesiolysis. Laparoscopic intervention is less traumatic, is accompanied by fewer postoperative complications and deaths, reduces the risk of adhesions, and allows for faster rehabilitation of patients. However, the use of laparoscopy for adhesive intestinal obstruction is possible in a limited number of patients. This is due to the high probability of intraoperative damage to the intestine, against the background of adhesions in the abdominal cavity and expansion of the loops of the small intestine, which occurs in 3-17% of patients.
* At the same time, the method, despite its low invasiveness, has limitations in its application due to the risk of bowel damage. Contraindications to its implementation are: the presence of more than 3 or more operations in the anamnesis, expansion of the small intestine more than 4 cm, intestinal necrosis or peritonitis
In other situations (multiple laparotomies, obstruction unrelated to the adhesive process, pronounced water-electrolyte disturbances, strangulation form of OSTKN with intestinal necrosis), surgical intervention is indicated - laparotomy.
##> Operation stages:
* Revision of the abdominal cavity, identification of the pathomorphological substrate of the obstruction. Taking the exudate of the abdominal cavity for bacteriological examination.
* Determination of the viability of the intestine in the area of the obstacle and determination of indications for its resection. When determining the indications for bowel resection, visual signs (color, peristalsis, pulsation and blood filling of the parietal vessels) are used, as well as the dynamics of these signs after the introduction of a local anesthetic solution into the mesentery and “warming” of the intestine with warm napkins moistened with saline (1,2). For an objective assessment of blood supply, it is also possible to use laser Doppler flowmetry, regional transillumination angiotensometry of the intramural vessels of the small intestine
* An operation is performed to disconnect adhesions, apply a bypass anastomosis or resect a section of the intestine (the type of intervention is selected individually). Detailed revision and determination of the site of obstruction are difficult due to a sharp swelling of the intestine. Therefore, during the operation, a long double-lumen nasointestinal probe with many openings for aspiration of the contents is inserted into the small intestine, if possible before the Bauhinia valve. In case of intestinal necrosis, resection is performed within the viable tissues, departing from the zone of necrosis in the adducting section by 30-40 cm, in the abducting section by 15-20 cm.The exception is resections near the Treitz’s ligament or the ileocecal angle, where it is allowed to limit these requirements with favorable visual characteristics intestines in the area of the proposed intersection. At the same time, control indicators are necessarily used: bleeding from the vessels of the wall at its intersection and the state of the mucous membrane.
* Considering the presence of a difference in the diameters of the small intestine, it is preferable to impose a small intestinal anastomosis “side to side”.
* Conducting a nasointestinal probe for decompression of the small intestine is mandatory. In most cases, drainage of the small intestine must be performed within 50-100 cm from the Treitz ligament with double-lumen probes for postoperative decompression and enetrotherapy. process, to ensure the frame function, or multiple iatrogenic damage to the intestinal wall for more adequate decompression. If it is impossible to pass the probe into the small intestine, due to a pronounced adhesion process in the upper floor of the abdominal cavity, an intraoperative endoscopic manual is used to install a nasojejunal drainage
Indisputable signs of nonviability of the intestine: dark coloration of the intestine, dull serous membrane, flabby intestinal wall, absence of pulsation of the mesenteric vessels, absence of intestinal motility.) It is resected.

How well did you know this?
1
Not at all
2
3
4
5
Perfectly
12
Q

A 76-year-old patient was admitted to the hospital on duty with complaints of severe intense pain in the upper abdomen and the umbilical region with irradiation to the lower back, left thigh, external genitalia, weakness, dizziness. From the anamnesis, it was revealed that during the last year he noted aching abdominal pain, unstable stool with a tendency to constipation, unpleasant sensations in the lumbar region, numbness of the feet. He has been suffering from hypertension for 30 years, blood pressure is 140/90 mmHg, increases to 180 mmHg are noted. Two years ago he suffered a myocardial infarction. During an objective examination, blood pressure is 90/60 mm Hg, pulse is 110 v min. The skin is pale. The abdomen is soft, not tense, peritoneal symptoms are negative. In the epigastric region, a pulsating volumetric formation is determined without clear contours, not displaced, sharply painful. Systolic noise is determined auscultatively over the formation.
Task:
1. What is the most likely diagnosis?
2. List the necessary additional examinations.
3. What are your treatment tactics?
4. Perform a differential diagnosis in this patient.
5. Surgical treatment options for this pathology.

A

1.Diagnosis: Chronic dissecting abdominal aortic aneurysm, type B according to Stanford
classification. Ruptured aneurysm
2. List the necessary additional examination
*ultrasound of Aorta
*pulse of the femoral vein or legs (palpation)
*Blood coagulation
* Abdominal CT scan. This painless test uses X-rays to create cross-sectional images of the structures inside the belly area. It’s used to create clear images of the aorta. An abdominal CT scan can also detect the size and shape of an aneurysm.
*
3. What are your treatment tactics
*hospitalisation
* The goal of abdominal aortic aneurysm treatment is to prevent an aneurysm from rupturing. Treatment may involve careful monitoring or surgery. Which treatment you have depends on the size of the aortic aneurysm and how fast it’s growing.
* Medical monitoring

A doctor might recommend this option, also called watchful waiting, if the abdominal aortic aneurysm is small and isn’t causing symptoms. Monitoring requires regular doctor’s checkups and imaging tests to determine if the aneurysm is growing and to manage other conditions, such as high blood pressure, that could worsen the aneurysm.
* surgical intervention
4. Perform a differential diagnosis in the patient
* bowel obstruction
* gastritis
*Acute pancreatitis
* mild pyleonephritis
5. Surgical options for the pathology
* Surgery to repair an abdominal aortic aneurysm is generally recommended if the aneurysm is 1.9 to 2.2 inches (4.8 to 5.6 centimeters) or larger, or if it’s growing quickly.
-Endovascular repair. This procedure is used most often to repair an abdominal aortic aneurysm. A surgeon inserts a thin, flexible tube (catheter) through an artery in the leg and gently guides it to the aorta. A metal mesh tube (graft) on the end of the catheter is placed at the site of the aneurysm, expanded and fastened in place. The graft strengthens the weakened section of the aorta to prevent rupture of the aneurysm.

Endovascular surgery isn’t an option for everyone with an abdominal aortic aneurysm. You and your doctor will discuss the best repair option for you. After endovascular surgery, you’ll need regular imaging tests to ensure that the grafted area isn’t leaking.
-Open abdominal surgery. This involves removing the damaged part of the aorta and replacing it with a graft, which is sewn into place. Full recovery may take a month or more.
*** Additional information

  1. The most likely diagnosis is an abdominal aortic aneurysm (AAA).
  2. The necessary additional examinations include abdominal ultrasound, CT scan, and blood tests (including kidney function tests).
  3. The treatment tactics may include blood pressure control, pain relief medication, and surgical intervention (endovascular repair or open surgical repair).
  4. Differential diagnosis may include acute pancreatitis, cholecystitis, and renal colic.
  5. Surgical treatment options for AAA include endovascular repair (EVAR) and open surgical repair (OSR). EVAR involves inserting a stent graft through a small incision in the groin and guiding it to the site of the aneurysm, while OSR involves replacing the damaged section of the aorta with a synthetic graft through an open incision in the abdomen.
How well did you know this?
1
Not at all
2
3
4
5
Perfectly
13
Q

A 54-year-old patient three days ago after fishing began to notice pain behind the sternum and epigastrium with irradiation in the back and left shoulder blade, increasing when swallowing, took nitroglycerin, without effect. The condition deteriorated sharply: the pain intensified, shortness of breath appeared, sharp weakness, dizziness. Upon admission, the condition is severe, pale, Blood pressure 100/40 mm Hg, pulse 110 v min., T = 39 degrees, crepitation in the supraclavicular region on the left, breathing is frequent, shallow, heart tones are deaf, rhythmic. During palpation, muscle tension in the epigastric region. On the review radiography of the chest organs, dilation and emphysema of the posterior mediastinum.
Task:
1. What is the most likely diagnosis?
2. List the necessary additional examinations.
3. What are your treatment tactics?
4. Perform a differential diagnosis in this patient.
5. Surgical treatment options for this pathology.

A
  1. Diagnosis: Esophageal perforation with a fish bone. Acute mediastinitis.
    2.list additional examinations
    *ECG
    *Gastroscopy
    *Endoscopy (can be used to remove the bone)
    *x-ray
    *C-scan
    *Blood analysis (coagulation test)
    3: what are your treatment tactics
    *Hospitalization of patient
    *surgical intervention
    *conservative treatment with the following
    -Intravenous fluids
    -Antibiotics (ceftraxone),penicillin,aminoglycosides
    -vitamins
  2. Differential diagnosis
    *ruptured aortic aneurysm
    *perforated gastric ulcer
    *strangulated diaphragmatic hernia
  3. Surgical options for this pathology
    *Separation operation (gastrostomy and esophagostomy)
    *surgical removal of foreign body
    *surgical drainage-To remove esophageal liquids if the inferior mediastinum drainage is done from the back with ultrasound
How well did you know this?
1
Not at all
2
3
4
5
Perfectly
14
Q

A 48-year-old man was taken to the emergency room with complaints of weakness, dizziness, moderate pain in the left hypochondrium. It is known from the anamnesis that 12 hours ago he fell down the stairs and hit his left rib arch. At first I felt satisfactory, and 2 hours ago, when walking, I suddenly felt weak, dizzy, fainting. The patient lies in a forced position on his left side. The skin is pale, the pulse is 120 beats. in min., blood pressure is 90/50 mm Hg. Palpation is marked by moderate tension of the abdominal wall muscles, dulling of sound in the left hypochondrium and lower abdomen.
Task:
1. What is the most likely diagnosis?
2. List the necessary additional examinations.
3. What are your treatment tactics?
4. Perform a differential diagnosis in this patient.
5. Surgical treatment options for this pathology.

A
  1. Diagnosis: Blunt abdominal trauma, double rupture of the spleen. Intra-abdominal bleeding. Hemor-
    rhagic shock, class 3
    2.List additional examinations
    *ultrasound
    *MRI
    *x-ray
    *complete blood counting :coagulation profile,hemoglobin, platelets count
  2. What are your treatment plans
    *Hospitalization
    *surgical intervention
    * conservative treatment
    4.perform a differential diagnosis
    *ruptured abdominal aorta
    *hemopneumothorax
    *Diaphragmatic injury
    5.surgical treatment options for this pathology
    *open laparostomy(if ruptured spleen occurred remove the spleen)
    ligation of blood vessel and suture of blood vessels
    **
    Additional information
  3. The most likely diagnosis is a splenic rupture due to the trauma suffered from the fall down the stairs.
  4. The necessary additional examinations include abdominal ultrasound, CT scan, and blood tests (hemoglobin, hematocrit, and platelets).
  5. The treatment tactics may include blood transfusions, fluid resuscitation, and surgical intervention (splenectomy).
  6. Differential diagnosis may include other causes of abdominal pain and weakness such as pancreatitis, acute cholecystitis, or aortic aneurysm rupture.
  7. Surgical treatment options for splenic rupture include splenectomy (removal of the spleen) or splenorrhaphy (repair of the spleen). In some cases, non-surgical management such as observation and blood transfusions may be considered.
How well did you know this?
1
Not at all
2
3
4
5
Perfectly
15
Q

A 62-year-old patient 5 hours ago suddenly had unbearable pains in her left hand and forearm, accompanied by weakness and numbness of the fingers. Three years ago, the patient underwent ONMC(acute cerebrovascular accident). The pulse is arrhythmic, 100 per minute. Auscultative diastolic murmur at the apex of the heart. Status localis: There is no asymmetry of the upper extremities during examination, active mobility in the wrist joint of the affected limb is limited, the strength of the left hand is significantly reduced compared to the right, the left hand is colder than the right. Pallor of the skin with a slight cyanotic tinge of the left hand, n/3 forearms, hyposthesia of the skin of the hand. In the axillary region, the pulsation is distinct, in the area of the elbow bend and distal parts of the forearm is not determined.
Task:
1. Name the most likely diagnosis.
2. What instrumental studies can confirm the diagnosis?
3. What medications does the patient need?
4. What are your treatment tactics?
5. What operations are performed for this disease?

A
  1. Diagnosis: Acute arterial insufficiency of the left upper limb of stage IIA
  2. The most likely diagnosis is acute arterial occlusion of the left upper limb.
  3. The instrumental studies that can confirm the diagnosis include Doppler ultrasound, angiography, and MRI.
  4. The patient needs anticoagulants and pain relief medication.
    * Heparin 5000 iu intravenously 20,000iu a day
    *Asprin 350 mg ,clopidogrel 350mg
  5. The treatment tactics may include thrombolysis (dissolving the blood clot), embolectomy (removing the blood clot), and revascularization (restoring blood flow to the affected limb).
  6. The operations that can be performed for this disease include embolectomy, bypass surgery, and angioplasty with stenting.
How well did you know this?
1
Not at all
2
3
4
5
Perfectly
16
Q

A 46-year-old patient, at the age of 20, suffered viral hepatitis B, subsequently, until now, she considered herself healthy. The reason for this hospitalization was a sudden general weakness, pallor, dizziness, a doubly black tar-like stool. Objectively: the state of moderate severity, blood pressure 90/60 mm Hg, pulse 96, skin pale, sclera subicteric, tongue dry, overlaid with a whitish coating. The abdomen is significantly enlarged in volume. Endoscopic examination revealed varicose veins of the lower thoracic and abdominal esophagus of the III st., continued bleeding from a defect in the wall of one of the veins. An. blood: Hb 90g/l, er. 2.9 x 1012/l, L 10.6 x 109/l, total bilirubin 25.5 mmol/l, straight 7.5 mmol/l, ALT 34ME/l (N10-35), AST - 58 IU/L.
Task:
1. Name the most likely diagnosis.
2. What additional studies should be carried out in the patient?
3. What medications does the patient need?
4. What are your treatment tactics?

A
  1. Diagnosis: Viral liver cirrhosis (hepatitis B). Complications: Portal hypertension. Ascites. Grade 3
    esophageal varices. Continuous bleeding. Hemorrhagic shock, class 1 Posthemorrhagic
    anemia of moderate severity.
  2. Additional examinations
    *Elastography
    *Fecal stool test
    *colonoscopy
    *complete blood count
    *Angiography
    *Imaging(CT scan abdominal)
  3. What medications does the patient need
    * octreotide 25-100mcg intravenously bolus of 50mcg)follow by continuous iv infusion of 25-50mcg per hour
    *Anticoagulants such as heparin 5000 IU 4 times a day =20000 IU
    *Tranexamic acid oral 500-650mg 3 times a day
    *Antibiotics 3 rd gen cephalosporins
  4. Treatment strategies
    *Endoscopic varicella Band ligation
    *octreotide ,Anti-coagulant,antibiotics,tranexamic acid
    *blood transfusions
    *iv fluids
    *liver transplant
17
Q

Patient S., aged 22, was admitted to the duty surgical hospital with complaints of dull, aching pain in the right iliac region, fever up to 38 C in the evenings, weakness, nausea, dry mouth. He considers himself ill for 5 days, when, as it seems to him, after eating poor-quality food, moderate epigastric pains appeared, which were accompanied by vomiting twice, which brought relief. Then the pain was localized in the right iliac region. To relieve pain, he took antispasmodics, analgesics. In view of the ongoing pain and the appearance of temperature, he turned to the local therapist, who sent him to the duty surgical hospital.
Objectively: Т=38.2 С, the tongue is densely coated with white coating, dryish. On the part of the chest organs, no pathology was detected. Heart rate 88 min. The abdomen is boat-shaped, the right lower half lags behind in the act of breathing, the navel is displaced somewhat to the right. On superficial palpation, there is tenderness in the right iliac region. With deep palpation, a painful, 8x10 cm in size, inactive formation is determined. Rovsing’s symptom is positive, Shchetkin-Blumberg’s symptom is negative. KLA: L- 9.4, OAM without pathology. Digital examination of the rectum: the ampoule is empty, the walls are painless, there is no overhang.
The task:
1. Name the most likely diagnosis.
2. Justify your diagnosis.
3. What additional studies should be performed on the patient?
4. What is your treatment strategy?
5. What complications can be expected in the patient?

A

1.Diagnosis: Acute appendicitis, appendicular infiltrate. Appendicular infiltrate and abscess.
2.justify your diagnosis
*clinical features such as
- high fever 38
-On superficial palpation, there is tenderness in the right iliac region
-Rovsing’s symptom is positive
3.what are the additional examinations
*complete blood count
-leukocytosis
-Biochemical blood analysis
-liver function test
-pancreatic enzymes
*instrumental diagnostic
-ultrasound
-CT scan
-x ray
4. Treatment strategies
*## Note: we don’t perform surgery in this patient because it’s appendicular infiltrate because adjacent organs will be damaged thus we hospitalize the patient and commence conservative treatment
- infusion therapy
-Antibiotics
-pain control ( tramadol)
-drainage T type
## planned appendectomy is performed 2-3 months later after patient hasn’t improved and incase of abscess opening and drainage of the abscess is performed
5:complications That can be expected
*abscess of the abdominal cavity

18
Q

Patient P., 38 years old, was admitted to the surgical hospital on duty as an ambulance with complaints of severe pain in the right hypochondrium, radiating into the right half of the chest, the back of the head, and the right upper limb. Ill for 48 hours. Previously, such attacks of pain were not observed. Objectively: T- 37.7 S, pale skin with a slight jaundice tinge, moderate ictericity of the sclera, soft palate. The tongue is moderately overlaid with a white coating. Chest organs without pathology. Abdomen: rounded shape, the right half lags behind in the act of breathing, with superficial palpation moderate tension and soreness in the right hypochondrium, an enlarged painful gallbladder is palpated. In the UAC: L - 10,2, E-5, P-7, C-70, M-2, L-18, ESR-18 mm / h. Bilirubin 120 microns / l, direct 80, indirect 40, alkaline phosphatase 120 E/ l, ultrasound: the liver is not enlarged, there are separate areas of increased echogenicity, intrahepatic bile ducts are expanded evenly in both lobes. The gallbladder is 85x35 mm in size, the wall is thickened, there are heterogeneous contents in the lumen, concretions from 0.1 to 0.3 cm. The choledoch is expanded to 1.5 cm . In its distal part, an echopositive formation with a size of 0.8 x 0.6 cm, with an acoustic shadow, is determined. EFGDS: mucus in the stomach, focal hyperemia of the mucosa. BDS is in the usual place, does not bulge out, bile does not enter the DPC.
Task:
1. Name the most likely diagnosis.
2. Justify your diagnosis.
3. Name the ultrasound signs of this disease
4. What additional studies should be carried out in the patient?
5. What is your treatment strategy?

A

1.Diagnosis: Cholelithiasis. Chronic calculous cholecystitis. Choledocholithiasis. Obstructive jaundice. Cholangitis.
2. Justify the diagnosis
*presence of jaundice
*Enlarged painful gallbladder is palpated , the wall is thickened with heterogeneous contents in the lumen,choledoch is expanded
*biochemical changes in bilirubin 120 increased and direct is 80 and indirect is 40
3.name the ultrasound studies of this disease
* there are separate areas of increased echogenicity, intrahepatic bile ducts are expanded evenly in both lobes. The gallbladder is 85x35 mm in size, the wall is thickened, there are heterogeneous contents in the lumen, concretions from 0.1 to 0.3 cm. The choledoch is expanded to 1.5 cm . In its distal part, an echopositive formation with a size of 0.8 x 0.6 cm, with an acoustic shadow, is determined.
4: what are the additional studies
*Ercp
*MRI constrast
*complete blood count
5. What is your treatment strategy
*Ercp therapy to remove the stones and lithoextraction from choleduch
*conservative treatment
- Intravenous fluids( 40mgl/kg)
-spasmolytics
*spasgan
*papaverine
Dotavarine
-Patient should not drink or feed
-Incase the stones cannot be removed perform Choledotomy
-Drainage (T-shaped)
-ursodeoxycholic acid
-cephalosporin (ceftriaxone)
**
Additional information

  1. The most likely diagnosis is acute cholecystitis with choledocholithiasis.
  2. The diagnosis is justified by the patient’s symptoms of severe pain in the right hypochondrium, jaundice, and an enlarged, painful gallbladder with thickened walls and concretions seen on ultrasound. The presence of an echopositive formation with an acoustic shadow in the distal part of the common bile duct suggests the presence of a stone.
  3. Ultrasound signs of acute cholecystitis include an enlarged, thick-walled gallbladder with pericholecystic fluid and concretions. Ultrasound signs of choledocholithiasis include an expanded common bile duct with an echopositive formation with an acoustic shadow seen in the distal part.
  4. Additional studies that should be carried out in the patient include blood tests (liver function tests, amylase), abdominal CT scan, and MRCP (magnetic resonance cholangiopancreatography).
  5. The treatment strategy may include antibiotics to treat the infection, pain relief medication, and endoscopic retrograde cholangiopancreatography (ERCP) to remove the stone from the common bile duct. If ERCP is not possible, surgical intervention (laparoscopic cholecystectomy with common bile duct exploration) may be necessary.
19
Q

A 58-year-old patient was admitted to the emergency room of the OKB 6 hours after the onset of the disease with complaints of unbearable, shingling pains in the upper half of the abdomen, vomiting that does not bring relief, weakness, dizziness, non-discharge of gases. The disease is associated with the use of fatty, overcooked food. In the anamnesis of GI, chronic cholecystitis for 4 years. An objective examination revealed a slight ictericity of the sclera, a dry overlaid tongue. Tachycardia up to 100 beats . in min. Palpation of the abdomen shows pronounced rigidity of the muscles in the epigastrium, absence of aortic pulsation, distension of the transverse colon.
Task:
1. Name the most likely diagnosis.
2. Which enzymes’ activity should be determined in this case?
3. Name the ultrasound signs of this disease
4. What are your treatment tactics?
5. What complications can be expected to develop in this patient?

A

1.Diagnosis: Primary diagnosis: Acute pancreatitis. Concomitant diagnosis: Cholelithiasis. Chronic calcu- lous cholecystitis. Choledocholithiasis?
## justification: A characteristic anamnesis - gallstone disease, chronic cholecystitis (the main cause in the population). With biliary pancreatitis (cholecystopancreatitis), pain occurs after a heavy meal.
The Mondor triad - pain, vomiting and flatulence - is pathognomonic for OP. The nature of pain, vomiting inherent in pancreatitis (not bringing relief) .
Due to paresis and swelling of the transverse colon or the presence of an infiltrate, it is not possible to clearly determine the pulsation of the abdominal aorta (Voskresensky’s symptom).
In the area of hypersensitivity, detected with superficial palpation, the muscles of the anterior abdominal wall are identified, which indicates the presence of pancreatogenic effusion, rich in enzymes, and the phenomena of pancreatogenic peritonitis.

2.which enzymes activity should be determined in this case
* Amylase 140-300 is high (4-fold increase)normal unit is 100 , lipase (2-fold increase indicates pancreatostasis, changes in concentration persist for a long time), Diastase (amylase) in urine, blood glucose (value more than 7 mmol / L - poor prognosis, large area of gland necrosis) …
ALP, transaminases (cholestasis); LDH (increases with damage to pancreatocytes);
A more specific study for the early diagnosis of pancreatitis is the determination of trypsin in the blood serum, α-chymotrypsin, elastase, carboxypeptidase, and especially phospholipase A, which plays a key role in the development of 4pancreatonecrosis. (complex analyzes - rarely used)
3. Name the ultrasound signs of this disease
* During the first days of the disease, ultrasound reveals an increase in the size of the pancreas, the unevenness of its contours, the echo heterogeneity of the structure, the presence of free fluid in the omental bursa or fluid formations in the retroperitoneal space and / or abdominal cavity. Accumulation of gas and fluid in swollen bowel loops. With necrosis of the pancreas, unsharply limited areas of reduced echogenicity or complete absence of an echo signal are revealed.

4.Treatment tactics
* Hospitalization. Cold + hunger + calm
*
* Correction of water and electrolyte balance - infusion therapy (1: 1 - colloids are needed to a greater extent for maintaining the BCC and binding enzymes in the blood) in the forced diuresis mode (after normalizing the BCC, we introduce diuretics) . For severe intoxication - plasmapheresis.
* Relief of pain and spasm of the sphincter of Oddi - m-anticholinergics (atropine, platifillin), antispasmodics (no-shpa).
* Octreotide (Sandostatin) is an inhibitor of basal and stimulated secretion of the pancreas, stomach and small intestine. The duration of therapy is 5-7 days.
* Proton pump blockers (Omeprazole) - to reduce acidity, secretion
* Extracorporeal detoxification (Multifiltration, Hemofiltration)
* If choledocholithiasis is found, EPST (endoscopic papillosphincterotomy)
* Drugs that suppress the secretion of the pancreas: Ringer’s solution-lactate + octreotide, contrikal, proton pump inhibitors (omeprazole 40-80 mg i.v.)
* From additions to basic therapy: Probing and aspiration of gastric contents. Infusion therapy in a volume of up to 40 ml per 1 kg of the patient’s body weight with forcing diuresis within 24-48 hours . For severe plasmapheresis / hemofiltration, antibiotic therapy for 3 days, epidural blockade, antiplatelet agents, nasogastric intubation for decompression and, if possible, nasogastrointestinal intubation for early enteral support .
5. What are the complications can be expected to develop in this patient
*diabetes mellitus
*peritonitis
*Shock—Hypovolemic and septic Respiratory failure and ARDS—Common in 7 days
*Septicaemia—Common after 7 days *Hypocalcaemia
* Disseminated intravascular coagulation (DIC)
*Acute renal failure
*Pancreatic pleural effusion (left sided 20%)
*Pancreatic pseudoaneurysm *Pancreatic ascites
*Colonic stricture
*Pseudocyst of pancreas
*Chronic pancreatitis
*Splenic vein thrombosis
*Abdominal compartment syndrome (ACS)
Pancreatic endocrine (15%) and exocrine (20%) insufficiency
**
Additional information

  1. The most likely diagnosis is acute pancreatitis.
  2. The enzymes that should be determined in this case include amylase and lipase.
  3. Ultrasound signs of acute pancreatitis include an enlarged, edematous pancreas with a hypoechoic appearance and peripancreatic fluid collections.
  4. The treatment tactics may include pain relief medication, fluid resuscitation, and nutritional support. In severe cases, hospitalization and intensive care may be necessary. The underlying cause of the pancreatitis (in this case, chronic cholecystitis) should also be addressed with appropriate treatment.
  5. Complications that can be expected to develop in this patient include pancreatic necrosis, pseudocysts, infected necrosis, and organ failure.
20
Q

Patient P., 53 years old, was admitted to the surgical hospital on duty with complaints: weakness, sticky cold sweat, vomiting of the type of “coffee grounds” in a volume of up to 1600ml, flashing “flies” in front of his eyes. The patient has a history of chronic gastritis with reduced secretion of gastric juice. Previous alcoholism. The skin is pale, the tongue is overlaid with a dirty gray coating, dryish. Pulse of low tension and filling, heart rate 100 per minute, blood pressure 100/60 mm Hg. With palpation, moderate pain in the epigastrium is noted. During endoscopic examination, a linear rupture of the mucosal-submucosal layer, up to 4 cm long, was found in the cardiac part of the stomach, from which scarlet blood flows down in a trickle. It is not possible to examine the stomach in other parts due to the presence of food masses mixed with brown blood in it.
Task:
1. What is the most likely diagnosis?
2. Justify your diagnosis.
3. Determine the degree of blood loss.
4. What are your further treatment tactics?
5. The causes of this disease?

A

1.Diagnosis: Mallory–Weiss syndrome, grade 2 (based on the classification of H. Bellman), continuous bleeding.haemorrhagic shock class III
2.justify the diagnosis
>Based on clinical findings
*coffee ground in volume 1600ml
*endoscopic exams result
*hypotension low blood pressure (100/60mmhg)
* increased heart rate 100
3. Determine the degree of blood loss
*Algover index is 1 :which corresponds approximately to a mild degree of blood loss( 750-1250) which 20% of BCC deficiency
4.Treatment tactics
##We first control the bleeding by the following methods
*Endoscopic clipping
*Injection or mechanical
*Thermal coagulation
* prescribe Tranexamic acid 500mg (Tranexamic acid (TXA) is a medication used to treat or prevent excessive blood loss from major trauma, postpartum bleeding, surgery, toothremoval, nosebleeds, and heavy menstruation
*Infusion Therapy crystalloids
*Entanzelat (4ml first dose or 2ml prescription three times
*Dicynene: Dicynene 500 mg Tablet is used in the treatment of bleeding. It is used to prevent or reduce bleeding in conditions like abnormal blood loss during periods and dysfunctional uterine bleeding. It also prevents or reduces bleeding during, before or after delicate surgeries.
*Aminocaprionic acid 50-100mg (Aminocaproic acid is an antifibrinolytic agent. It is used to treat serious bleeding conditions )+vicasone(vitamin k 12-42 hours )
*calcium chloride 10%
*PPi therapy is also included
*Fresh frozen plasma (1-2 doses)
## note surgery is the last option to suture the bleeding
5 . The causes of this disease
*Alcohol is primary cause
* violent coughing or vomiting.

21
Q

Patient H., 32 years old, was operated on urgently for phlegmonous appendicitis complicated by local serous peritonitis. The wound was sutured with a micro-irrigator. On the 5th day after the operation, the patient had indeterminate pain in the area of surgical intervention. By the evening, the pain intensified, began to have a cramping character, accompanied by vomiting. The surgeon on duty during the examination found a soft abdomen, moderately painful in the area of surgery, there are no symptoms of irritation of the peritoneum.
UAC: L - 7,4. When performing an overview radiography of the abdominal cavity, Cloiber bowls were found along the course of the small intestine.
Task:
1. What is the most likely diagnosis?
2. Justify your diagnosis.
3. What is the evidence of the X-ray data obtained?
4. What is your further treatment tactics?
5. Prevention of this disease?

A

1.Diagnosis: Early intestinal adhesions with obstruction.

  1. The most likely diagnosis is small bowel obstruction.
  2. The diagnosis is supported by the patient’s symptoms of abdominal pain and vomiting after surgery, as well as the presence of Cloiber bowls (dilated loops of small intestine) seen on the X-ray.
  3. The evidence of the X-ray data obtained is the presence of Cloiber bowls, which are dilated loops of small intestine that are characteristic of small bowel obstruction.
  4. The further treatment tactics may
    ##relaparotomy, revision of abdominal organs, adhesiolysis and restoration of intestinal patency, assessment of the viability ofthe strangled intestine.

Also nasogastric tube insertion to relieve pressure in the bowel, fluid and electrolyte replacement therapy, and surgical intervention if conservative measures are unsuccessful. The underlying cause of the obstruction (such as adhesions from the previous surgery) should also be addressed.
5. Prevention of small bowel *obstruction includes proper surgical technique to minimize adhesion formation, early mobilization after surgery, and avoiding constipation. Patients should also seek medical attention promptly if they experience symptoms of abdominal pain or vomiting after surgery.
* the use of Mesogel, drainage ofthe abdominal cavity according to indications, early activation ofpatients and early enteral nutrition in order torestore the motor evacuation function ofthe gastrointestinal tract

22
Q

A 43-year-old patient came to the surgical clinic complaining of pain in the epigastrium with a spread to the iliac region.
I woke up in the morning from a sharp pain in the epigastrium. Taking no-shpa and almagel slightly reduced the pain syndrome, but there was a sharp weakness, nausea. He is on the dispensary register for peptic ulcer disease.
On examination: the condition of the patient of moderate severity. The position is forced - he lies with his legs tucked up to his stomach. In the back position, the pain increases. Pathology on the part of the chest organs was not revealed. The abdomen is tense in all departments, but more in the epigastrium, there is also sharp soreness. There is no hepatic dullness. Mace-Robson’s S-m is doubtful. Symptoms of Ortner, Georgievsky-Mussy are negative. Deep palpation of the abdomen is impossible. The prescribed injection of atropine, diphenhydramine and analgin did not give the desired effect.
Task:
1. What is the most likely diagnosis?
2. Justify your diagnosis.
3. What studies should be carried out by the patient in the emergency department?
4. What are your further treatment tactics?
5. What are your actions when the patient categorically refuses surgery?

A

1: Diagnosis: peptic ulcer Perforated ulcer, peritonitis.
2.justify the diagnosis
*Based on clinical findings: pain in the epigastrium with a spread to the iliac region.,
* He is on the dispensary register for peptic ulcer disease.
* the condition of the patient of moderate severity. The position is forced - he lies with his legs tucked up to his stomach. In the back position, the pain increases.
* Deep palpation of the abdomen is impossible. The prescribed injection of atropine, diphenhydramine and analgin did not give the desired effect.
3.what studies should be carried out by the patient in the emergency department
* Chest X-ray with abdomen in erect posture (plain X-ray): Shows gas under diap hragm in 70% of cases. In 30% of cases, there is no gas under diaphragm. It may be due to, either the gas leak is less than 1 ml or due to previous surgery causing adhesions between liver and diaphragm, or sealed peptic ulcer. (Chilaiditi’s syndrome is the interposition of the colon in front and above the liver. It is common in children and elderly. It may be mistaken for gas under diaphragm in plain X-ray abdomen).
* Ultrasound abdomen shows free fluid and often gas.
* Blood urea, serum creatinine, total count, electro lytes, are helpful.
* CT scan abdomen is very sensitive investigation whenever there is absence of gas under diaphragm. It rules out other conditions like pancreatitis. Gastrograffin upper GI study also confirms the perforation.
4: what are your further tactics
* Surgery: Emergency laparotomy through upper midline incision is done. All infected fluid is sucked out. Perforation is identified and closed with interrupted, horizontal sutures using either silk or vicryl. Omental patch is placed before suturing—it is called as Rosoe-Graham Operation. Because of its adhesion property it seals perforation; good vascularity and lymphatics promote the healing effectively.
* Patient is hospitalized .
* IV fluids—Ringer lactate, normal saline, dextrose saline.
*Antibiotics—Cefotaxime, metronidazole, amikacin.
* Catheterisation.
*Ryle’s tube aspiration.
5. What are your actions when patient categorically refuses surgery
*conservative treatment
-Patient is advised admission.
-IV fluids—Ringer lactate, normal saline, dextrose saline.
-Antibiotics—Cefotaxime, metronidazole, amikacin. *Catheterisation.
*Ryle’s tube aspiration.

23
Q

A 28-year-old man came to the hospital with complaints of pain in the left half of the abdomen, nausea, single vomiting, tumor-like formation in the scrotum.
I was sick for about 7 hours. It is known from the anamnesis that similar pains have occurred before, a rounded formation came out into the scrotum, but it was set on its own. This has not happened now.
Task:
1. What is the most likely diagnosis?
2. What is your further treatment tactics?
3. Anatomy of the inguinal canal.
4. Methods of operations for inguinal hernias.
5. The volume of resection of the necrotic intestine in case of hernia infringement.

A
  1. Diagnosis: Incarcerated inguinal scrotal hernia on the left. Acute intestinal obstruction.
    1. The most likely diagnosis is inguinal hernia with incarceration or strangulation.
  2. The further treatment tactics may include surgical intervention to reduce the hernia and repair the defect in the inguinal canal.
  3. The inguinal canal is an oblique passage through the lower abdominal wall that contains the spermatic cord in men and the round ligament of the uterus in women. It is bounded by the inguinal ligament, the inferior epigastric vessels, the transversus abdominis muscle, the internal oblique muscle, and the external oblique muscle.
  4. The methods of operations for inguinal hernias include open hernia repair (using a small incision in the groin to access and repair the hernia) and laparoscopic hernia repair (using a laparoscope to repair the hernia from within the abdomen).
  5. The volume of resection of the necrotic intestine in case of hernia infringement depends on the extent of the necrosis and the location of the hernia. In some cases, a small segment of the intestine may be resected, while in other cases, a larger portion of the intestine may need to be removed. The decision on the extent of resection is made based on the surgeon’s assessment of the situation during the operation.
    * It is necessary to assess the viability of the pinched loop ofthe intestine. With necrosis of the pinched loop of the small intestine, resection of 30-40 cm of the adductor section above necrosis and 15-20 mc of hte diverting section (below necrosis.
24
Q

Patient H., 32 years old, turned to the district therapist with complaints of weight loss, vomiting with stagnant gastric contents, convulsions, swelling in the epigastric region. From the anamnesis, it was found out that the patient is periodically disturbed for a long time by severe epigastric pain, which decreases after taking Maalox. The patient was treated independently, did not seek medical help.
Objectively: a patient with low nutrition. The tongue is dry, overlaid with a whitish coating, there is an unpleasant, sour smell from the mouth. The abdomen participates in the act of breathing, painless with palpation. In the UAC: L - 8,2, E-4, P-7, S-70, M-2, L-18, ESR-18 mm/h. In the biochemical analysis of blood: bilirubin 12 microns / l, direct 2, indirect 10, alkaline phosphatase 120 E / l, total protein -56 g /l
Task:
1. What is the most likely diagnosis?
2. Justify your diagnosis.
3. What additional studies will confirm or refute your diagnosis?
4. What are your further treatment tactics?
5. Prevention of this disease?

A

1.diagnosis: Peptic and duodenal ulcer disease. Uncompensated pyloroduodenal stenosis.
2. 1. The most likely diagnosis is peptic ulcer disease with complications such as gastric outlet obstruction and electrolyte imbalances.
2. The diagnosis is supported by the patient’s symptoms of weight loss, vomiting, and epigastric pain that is relieved by antacids. The presence of swelling in the epigastric region suggests possible gastric outlet obstruction.
3. Additional studies that can confirm or refute the diagnosis include upper gastrointestinal endoscopy, abdominal CT scan, and stool tests for Helicobacter pylori (a bacteria that is commonly associated with peptic ulcers).
4. The further treatment tactics may include proton pump inhibitors to reduce stomach acid production, antibiotics to eradicate H. pylori infection (if present), and surgery to relieve gastric outlet obstruction if necessary. The patient may also need electrolyte replacement therapy to correct any imbalances.
*pyloriplasty as surgical intervention and bilroth 1 or bilroth 2
5. Prevention of peptic ulcer disease includes,timely detection of the disease and early treatment also avoiding triggers such as spicy and acidic foods, reducing stress, and avoiding nonsteroidal anti-inflammatory drugs (NSAIDs) unless prescribed by a doctor. Treatment of H. pylori infection can also help prevent peptic ulcers.

25
Q

A taxi driver was delivered to the surgical clinic 8 hours after the illness. During a long trip, he felt a sharp pain in the epigastrium, the soda at hand and the gastropharm(antacids )and analgin( metamizole,is an analgesic)bought at the pharmacy on the way, did not give relief as it used to happen. The pain that initially arose in the epigastrium began to spread throughout the right half of the abdomen. At each point of the car, the pain intensified. Over time, the pain acquired a dull character, but spread throughout the abdomen and more in the lower parts.
On examination: the skin is pale. The tongue is dry, overlaid with a white coating. Pulse 76 per minute, rhythmic. Blood pressure 120/89 mm Hg. From the side of the lungs and heart pathology is not determined. The abdomen is well-shaped, moderately tense in all departments. Hepatic stupidity is preserved. Percussion reveals a painful point in the epigastrium. In sloping places, blunting. Symptoms of irritation of the peritoneum in all departments are weakly positive.
When performing radiography of the abdominal cavity , the following picture was obtained:
Task:
1. What is the most likely diagnosis?
2. Justify your diagnosis.
3. What is the evidence of the X-ray data obtained?
4. What are your further treatment tactics?
5. What options for surgical treatment exist for this pathology?

A

1:Diagnosis: Acute ulcer with perforation.
2.justify the diagnosis
*clinical symptoms like pain in the epigastric
*peritoneum irritation
*percussion reveals a painful point in the epigastrium
3. Evidence of x ray reveals what ?
*Gas beneath the diaphragm in the right dome
4. What are your further treatment tactics
*Hospitalization
* surgical treatment:Resection of the stomach can be performed within 6 hours and use omentum patch has the best option to cover resected part
*Exercision of the duodenal ulcer with pyloric plasty
*patient can only begin feeding (drink) in 2-3 days
*crystalloids fluids 3liters per day
*Antispasmodics
*pain killers (morphine 2days), tramadol,ketarol.
*Antibiotics ( Metronidazole, clarithromycin
- For peritonitis treat with cephalosporin 3rd generation ceftraxione